Intersection of balls in $l^{infty}$












1












$begingroup$


Must it be the case that the intersection of any two closed balls in $l^{infty}$ either is empty, a single point, or contains a non-empty open ball?










share|cite|improve this question









$endgroup$








  • 5




    $begingroup$
    Hint: consider the closed balls of radius 1 centered at the points $(0,0,0,0,dots)$ and $(2,0,0,0,dots)$.
    $endgroup$
    – Nate Eldredge
    Dec 24 '18 at 1:07
















1












$begingroup$


Must it be the case that the intersection of any two closed balls in $l^{infty}$ either is empty, a single point, or contains a non-empty open ball?










share|cite|improve this question









$endgroup$








  • 5




    $begingroup$
    Hint: consider the closed balls of radius 1 centered at the points $(0,0,0,0,dots)$ and $(2,0,0,0,dots)$.
    $endgroup$
    – Nate Eldredge
    Dec 24 '18 at 1:07














1












1








1


1



$begingroup$


Must it be the case that the intersection of any two closed balls in $l^{infty}$ either is empty, a single point, or contains a non-empty open ball?










share|cite|improve this question









$endgroup$




Must it be the case that the intersection of any two closed balls in $l^{infty}$ either is empty, a single point, or contains a non-empty open ball?







real-analysis general-topology functional-analysis






share|cite|improve this question













share|cite|improve this question











share|cite|improve this question




share|cite|improve this question










asked Dec 24 '18 at 0:04









Andrew LizarragaAndrew Lizarraga

4215




4215








  • 5




    $begingroup$
    Hint: consider the closed balls of radius 1 centered at the points $(0,0,0,0,dots)$ and $(2,0,0,0,dots)$.
    $endgroup$
    – Nate Eldredge
    Dec 24 '18 at 1:07














  • 5




    $begingroup$
    Hint: consider the closed balls of radius 1 centered at the points $(0,0,0,0,dots)$ and $(2,0,0,0,dots)$.
    $endgroup$
    – Nate Eldredge
    Dec 24 '18 at 1:07








5




5




$begingroup$
Hint: consider the closed balls of radius 1 centered at the points $(0,0,0,0,dots)$ and $(2,0,0,0,dots)$.
$endgroup$
– Nate Eldredge
Dec 24 '18 at 1:07




$begingroup$
Hint: consider the closed balls of radius 1 centered at the points $(0,0,0,0,dots)$ and $(2,0,0,0,dots)$.
$endgroup$
– Nate Eldredge
Dec 24 '18 at 1:07










1 Answer
1






active

oldest

votes


















1












$begingroup$

To expand Nate's comment into a full answer: let us set $x_1:=(0,0,0,ldots )$ and $x_2:=(2,0,0,0,ldots )$ and then $$ bar{B_i}(x_i,1):={ yin ell_infty : |y-x|_infty leq 1 }$$
Since $|x|_infty = max_{iin {mathbb N}} |x_i|$ we see that the point $(1,0,0,0,ldots )$ will lie in both $bar{B_1}$ and $bar{B_2}$. In fact, you should be easily able to show that
$$bar{B_1} cap bar{B_2} = { (1, y_1, y_2, y_3, ldots ) : |y_i|leq 1 : forall i in {mathbb N} } $$
So: the intersection is




  1. clearly non-empty

  2. contains infinitely many points

  3. does not contain an open ball (because the first co-ordinate of all the points in the intersection is forced to be $1$.






share|cite|improve this answer









$endgroup$













    Your Answer





    StackExchange.ifUsing("editor", function () {
    return StackExchange.using("mathjaxEditing", function () {
    StackExchange.MarkdownEditor.creationCallbacks.add(function (editor, postfix) {
    StackExchange.mathjaxEditing.prepareWmdForMathJax(editor, postfix, [["$", "$"], ["\\(","\\)"]]);
    });
    });
    }, "mathjax-editing");

    StackExchange.ready(function() {
    var channelOptions = {
    tags: "".split(" "),
    id: "69"
    };
    initTagRenderer("".split(" "), "".split(" "), channelOptions);

    StackExchange.using("externalEditor", function() {
    // Have to fire editor after snippets, if snippets enabled
    if (StackExchange.settings.snippets.snippetsEnabled) {
    StackExchange.using("snippets", function() {
    createEditor();
    });
    }
    else {
    createEditor();
    }
    });

    function createEditor() {
    StackExchange.prepareEditor({
    heartbeatType: 'answer',
    autoActivateHeartbeat: false,
    convertImagesToLinks: true,
    noModals: true,
    showLowRepImageUploadWarning: true,
    reputationToPostImages: 10,
    bindNavPrevention: true,
    postfix: "",
    imageUploader: {
    brandingHtml: "Powered by u003ca class="icon-imgur-white" href="https://imgur.com/"u003eu003c/au003e",
    contentPolicyHtml: "User contributions licensed under u003ca href="https://creativecommons.org/licenses/by-sa/3.0/"u003ecc by-sa 3.0 with attribution requiredu003c/au003e u003ca href="https://stackoverflow.com/legal/content-policy"u003e(content policy)u003c/au003e",
    allowUrls: true
    },
    noCode: true, onDemand: true,
    discardSelector: ".discard-answer"
    ,immediatelyShowMarkdownHelp:true
    });


    }
    });














    draft saved

    draft discarded


















    StackExchange.ready(
    function () {
    StackExchange.openid.initPostLogin('.new-post-login', 'https%3a%2f%2fmath.stackexchange.com%2fquestions%2f3050821%2fintersection-of-balls-in-l-infty%23new-answer', 'question_page');
    }
    );

    Post as a guest















    Required, but never shown

























    1 Answer
    1






    active

    oldest

    votes








    1 Answer
    1






    active

    oldest

    votes









    active

    oldest

    votes






    active

    oldest

    votes









    1












    $begingroup$

    To expand Nate's comment into a full answer: let us set $x_1:=(0,0,0,ldots )$ and $x_2:=(2,0,0,0,ldots )$ and then $$ bar{B_i}(x_i,1):={ yin ell_infty : |y-x|_infty leq 1 }$$
    Since $|x|_infty = max_{iin {mathbb N}} |x_i|$ we see that the point $(1,0,0,0,ldots )$ will lie in both $bar{B_1}$ and $bar{B_2}$. In fact, you should be easily able to show that
    $$bar{B_1} cap bar{B_2} = { (1, y_1, y_2, y_3, ldots ) : |y_i|leq 1 : forall i in {mathbb N} } $$
    So: the intersection is




    1. clearly non-empty

    2. contains infinitely many points

    3. does not contain an open ball (because the first co-ordinate of all the points in the intersection is forced to be $1$.






    share|cite|improve this answer









    $endgroup$


















      1












      $begingroup$

      To expand Nate's comment into a full answer: let us set $x_1:=(0,0,0,ldots )$ and $x_2:=(2,0,0,0,ldots )$ and then $$ bar{B_i}(x_i,1):={ yin ell_infty : |y-x|_infty leq 1 }$$
      Since $|x|_infty = max_{iin {mathbb N}} |x_i|$ we see that the point $(1,0,0,0,ldots )$ will lie in both $bar{B_1}$ and $bar{B_2}$. In fact, you should be easily able to show that
      $$bar{B_1} cap bar{B_2} = { (1, y_1, y_2, y_3, ldots ) : |y_i|leq 1 : forall i in {mathbb N} } $$
      So: the intersection is




      1. clearly non-empty

      2. contains infinitely many points

      3. does not contain an open ball (because the first co-ordinate of all the points in the intersection is forced to be $1$.






      share|cite|improve this answer









      $endgroup$
















        1












        1








        1





        $begingroup$

        To expand Nate's comment into a full answer: let us set $x_1:=(0,0,0,ldots )$ and $x_2:=(2,0,0,0,ldots )$ and then $$ bar{B_i}(x_i,1):={ yin ell_infty : |y-x|_infty leq 1 }$$
        Since $|x|_infty = max_{iin {mathbb N}} |x_i|$ we see that the point $(1,0,0,0,ldots )$ will lie in both $bar{B_1}$ and $bar{B_2}$. In fact, you should be easily able to show that
        $$bar{B_1} cap bar{B_2} = { (1, y_1, y_2, y_3, ldots ) : |y_i|leq 1 : forall i in {mathbb N} } $$
        So: the intersection is




        1. clearly non-empty

        2. contains infinitely many points

        3. does not contain an open ball (because the first co-ordinate of all the points in the intersection is forced to be $1$.






        share|cite|improve this answer









        $endgroup$



        To expand Nate's comment into a full answer: let us set $x_1:=(0,0,0,ldots )$ and $x_2:=(2,0,0,0,ldots )$ and then $$ bar{B_i}(x_i,1):={ yin ell_infty : |y-x|_infty leq 1 }$$
        Since $|x|_infty = max_{iin {mathbb N}} |x_i|$ we see that the point $(1,0,0,0,ldots )$ will lie in both $bar{B_1}$ and $bar{B_2}$. In fact, you should be easily able to show that
        $$bar{B_1} cap bar{B_2} = { (1, y_1, y_2, y_3, ldots ) : |y_i|leq 1 : forall i in {mathbb N} } $$
        So: the intersection is




        1. clearly non-empty

        2. contains infinitely many points

        3. does not contain an open ball (because the first co-ordinate of all the points in the intersection is forced to be $1$.







        share|cite|improve this answer












        share|cite|improve this answer



        share|cite|improve this answer










        answered Dec 24 '18 at 15:37









        postmortespostmortes

        2,04121119




        2,04121119






























            draft saved

            draft discarded




















































            Thanks for contributing an answer to Mathematics Stack Exchange!


            • Please be sure to answer the question. Provide details and share your research!

            But avoid



            • Asking for help, clarification, or responding to other answers.

            • Making statements based on opinion; back them up with references or personal experience.


            Use MathJax to format equations. MathJax reference.


            To learn more, see our tips on writing great answers.




            draft saved


            draft discarded














            StackExchange.ready(
            function () {
            StackExchange.openid.initPostLogin('.new-post-login', 'https%3a%2f%2fmath.stackexchange.com%2fquestions%2f3050821%2fintersection-of-balls-in-l-infty%23new-answer', 'question_page');
            }
            );

            Post as a guest















            Required, but never shown





















































            Required, but never shown














            Required, but never shown












            Required, but never shown







            Required, but never shown

































            Required, but never shown














            Required, but never shown












            Required, but never shown







            Required, but never shown







            Popular posts from this blog

            Quarter-circle Tiles

            build a pushdown automaton that recognizes the reverse language of a given pushdown automaton?

            Mont Emei